Prince Exam 2

Réussis tes devoirs et examens dès maintenant avec Quizwiz!

Which of the following substrates would likely react with GSH in the presence of GST? "GSTs accomplish this by catalyzing conjugation reactions between the tripeptide glutathione (GSH) and a variety of electrophilic substrates, including aliphatic halides, nitroesters, and thiocyanates." Diagram on next page, don't peep answer Question -> how should I approach answering this? A. II only B. III only C. I and III only D. II and III only

*NOTICE THAT YOU HAVE TO LOOK BACK INTO PASSAGE TO GET ANSWER* Item I is false. The passages mentions that GSTs conjugate GSH with "aliphatic halides, nitroesters, and thiocyanates." The passage does not indicate that a hydrocarbon with a mesyl group reacts with GSH (Choice C is wrong). Item II is false. Although this compound contains an ester functional group, it does not contain a nitro (-NO2) group and is not a nitroester (choices A and D are wrong; choice B is correct). Item III is true. The prefix thio indicates the presence of a sulfur group and cyano is indicative of a CN functional group. Answer is B

1000 L = __m^3 *1L = ___ m^3 1L=____cm^3 1mL = .... =....

1000 L = 1m^3 1L = 10^-3 m^3 1L = 1000cm^3 1mL = 1 cm^3 = 1cc

The first daughter nucleus in the 238U decay series is thorium-234. If the next four decays are two β- and two α decays, what is the resulting radionuclide? A. Radium-226 B. Polonium-226 C. Radon-228 D. Thorium-230

A. Each β- decay increases the number of protons by 1, but leaves the total number of protons and neutrons unchanged. Each α decay decreases the number of protons by 2 and decreases the total number of protons and neutrons by 4. Thus, after two β- and two α decays, thorium-234 becomes radium-226:

A healthy woman has a respiratory minute volume (the amount of air inhaled and exhaled in one minute) of about 6 L. Suppose the breathing tube she uses has a cross-sectional area of 1 cm2. What is the average flow speed through the tube? A. 1 m/s B. 3.1 m/s C. 100 m/s D. 310 m/s

A. The volume flow rate (volume of air flowing per second) is given by the equation f = Av, where A is the cross-sectional area of the cylinder and v is the average speed of the flow. The flow rate in this case is f = 6 L/min × (1 m3 / 1000 L) × (1 min / 60 s) = 1 × 10-4 m3/s. The cross-sectional area is 1 cm2 × (1 m / 100 cm)2 = 10-4 m2. Thus v = f / A = (1 × 10-4 m3/s) / (10-4 m2) = 1 m/s.

If 99% of the H+ ions in a solution of pH 1.3 were neutralized, what would be the final pH? A. 0.3 B. 3.3 C. 5.3 D. 6.9

B. If only 1% = 1/100 of the H+ ions remained, then the number of H+ ions would drop by a factor of 102. Therefore, the pH would increase by log(102) = 2 units. In this case, the pH would rise from 1.3 to 1.3 + 2 = 3.3.

Which of the following best justifies the use of DNA, as opposed to RNA, for generating aptamers? The aptamers are single-stranded and capable of forming a tremendous variety of three-dimensional shapes. A. DNA is double-stranded. B. RNA is single-stranded. C. DNA lacks a 2' hydroxyl group. D. RNA contains uracil instead of thymine.

C. As aptamers are single-stranded, the stability of the oligonucleotides is important. DNA lacks the 2' hydroxyl group on RNA that leads to self-cleavage, making DNA more stable than RNA and better suited as an aptamer (choice C is correct). According to the passage, the DNA aptamers used are single stranded, making the strandedness of the original nucleotide irrelevant (choices A and B are wrong). Uracil and thymine are essentially identical except for a methyl group but that would not significantly affect the binding capability of the oligonucleotides (choice D is wrong).

Hydroxyurea is a medication that can increase the concentration of HbF made in an adult. Would this medication be useful to a patient with sickle cell disease? HbF does not have beta chain but sickle cell rbc do. A. No, because the defect in sickle cell disease affects the β-chain of hemoglobin and consequently, HbF will sickle also. B. No, because unlike HbA, the HbF would not carry oxygen. C. Yes, because the HbF, which can carry oxygen, will not sickle. D. Yes, because HbF will carry more oxygen than HbA.

C. HbF actually has a greater affinity for oxygen than HbA does (choice B is incorrect). This is the way that fetal hemoglobin can extract oxygen from the maternal circulation through the placenta. Since HbF does not contain β-chains, it is not subject to sickling (choice C is correct and choice A is incorrect). HbF can only carry four molecules of oxygen, the same as HbA (choice is incorrect).

Similarities that exist between protists and bacteria include all of the following except: A. Organelles B. Double-stranded DNA C. Unicellularity D. Flagellar motility

Protists are like algae A. Bacteria do not have organelles while protists exhibit the organelles typical of animal cells (answer choice A is correct). While bacterial store genetic information in circular DNA and protists have linear chromosomes, both forms of DNA are double-stranded and, thus, this is a similarity between bacteria and protists (answer choice B is incorrect). Bacteria are unicellular and protists typically are, as well (answer choice C is wrong). Some protists exhibit amoeboid movement but both protists and bacteria often exhibit flagellar movement (answer choice D is wrong).

If the alcohol in the Pitot tube were replaced with water, then the height difference of the liquid in the U-tube for a given air speed would be: (given equation in passage v = square root of (2gyPliquid/Pgas)) A. smaller. B. the same. C. greater. D. impossible to predict.

Strategy for finding relation, use given or know equation and put constants/variables they mention on opposite side of equation so see if direct or inverse relationship A. The equation given in the passage can be rearranged to show that y = v2ρgas / (2gρliquid) → y ∝ 1 / ρliquid. Since ρwater is greater than ρalcohol ρliquid increases causing y to decrease.

Based on the table above, which one of the following species is the strongest reducing agent? A. Ba B. Cu+ C. PbSO4 D. F-

The strongest reducing agent is the species that it most easily oxidized. Choice B can be eliminated since the reduction of Cu+ is spontaneous (since E° > 0). Choice D can also be eliminated because the oxidation of F- is nonspontaneous. The oxidation of Ba (to Ba2+) is spontaneous (E° = +2.91 V) and has a higher voltage than the (magnitude of) the voltage for the reduction of PbSO4, so barium is the strongest reducing agent here. The answer is A.

Oil drop, dropped above plates (see diagram on next side but don't look at answer) Which of the following equations gives the voltage V necessary to maintain an oil droplet of mass m and charge -q suspended in the region between Plates A and B, which are separated by a distance d? A. V = mgd / q B. V = mg / (qd) C. V = gd / (mg) D. V = q / (mgd)

The voltage, V, and the electric field strength, E, between Plates A and B are related by the equation V = Ed, where d is the distance between the plates. The oil drop is suspended in the region between the plates when qE = mg (see the prior question), or, in terms of V, when q(V / d) = mg. Solving this last equation for V gives V = mgd / q. Answer A

PLX 4720 and PLX 4032 would be expected to bind with greatest affinity to an enzyme active site containing a large number of which of the following residues? A. V and L B. D and N C. D and K D. K and S So as soon as you see a question like this, what should you look for/think about?

Think about if there was any mention of the active site being hydrophobic or hydrophilic. The passage states that the substances are inhibitors of the B-Raf ATP binding site, which was elsewhere in the passage described as hydrophobic. Given this, PLX 4720 and PLX 4032 would be expected to bind most strongly to active sites enriched in hydrophobic residues, such as valine (V) and leucine (L). Aspartic acid (D) is an acidic amino acid and will be negatively charged in the body, so choices B and C can be eliminated. Lysine (K) is a basic amino acid and will be positively charged, making choice D incorrect. Answer is A

Inclusive fitness

the ability of an individual organism to pass on its genes to the next generation, taking into account the shared genes passed on by the organism's close relatives.

A dilute multi-component aqueous solution was prepared containing equimolar amounts of AgI, AgCN, AgBr, and AgCl. Dropwise addition of concentrated AgNO3 would cause which of these compounds to precipitate first? A. AgI B. AgCN C. AgBr D. AgCl

A. The AgNO3 will dissolve as it is added to each AgX solution, giving Ag+ and NO3- ions in solution. The concentration of Ag+ increases until the ion product, [Ag+ ][X-], (you only include these two in the equation bc AgI is a solid and you don't include solids in Keq equation) is equal to the Ksp for AgX, at which point AgX begins to precipitate. Thus, the compound that would precipitate first is the one with the lowest Ksp. Silver iodide, AgI, has the lowest Ksp value in the table. -Marky explanation is that since AgI dissolved the least (lowest Ksp) and you are basically adding the common ion to solution, this grealy favors reactants so it precipitates

*When asked to indicate what contributed to their desirability ratings, none of the daters mentioned the issue of whether the individual whom they were rating approached them or remained stationary. Moreover, all of the daters instead focused upon the individual's perceived personality and character traits. In light of the findings stated in the passage and the data in Figure 1, this might be an example of: (It was a speed dating experiment, where men and women approached others who were either sitting or standing) A. a self-fulfilling prophecy. B. stereotyping. C. socialized identity formation. D. the fundamental attribution error.

D. The fundamental attribution error is the tendency to interpret others' behavior in terms of personal attributes, as opposed to situational factors. In this case, the daters may have tended to interpret what they perceived as desirable or undesirable behavior during the speed dating interactions as dispositional, rather than simply the result of the structured situation (choice D is correct). A self-fulfilling prophecy occurs when the expectation of an event leads directly or indirectly to that event happening (choice A is wrong). Stereotyping involves viewing members of a social group in a distorted and often negative way based on preconceptions about the characteristics of that group (choice B is wrong). Socialized identity formation is the establishment of a distinct personality based on interactions with others (choice C is wrong).

*What do Western, Southern, and Northern blots do?

Detection of a protein can be done using a Western blot. Northern blot is designed to detect specific RNA sequences, and a Southern blot is designed to detect specific DNA sequences.

Which of the following would be a reasonable explanation for the discrepancies in cortisol patterns between the Black and Hispanic High CSS groups? Graphs on next side A. A smaller percentage of Hispanic participants experienced a high degree of stress than did Black participants. B. Blacks' physiological response to stress is more effective than that of Hispanics. C. Pregnancy hormones improve HPA axis functioning. D. Social support systems are stronger in Hispanic communities than in Black communities.

The fact that Hispanics in the high CSS group did not show high cortisol levels, while the Blacks in the high CSS group did show elevated levels, suggests that the same amount of psychosocial stress impacted the two groups differently. One possible explanation is that Hispanic communities offer stronger social support, which helps to mediate the impact of stress (choice D is correct). While it is true that a smaller percentage of Hispanic participants experienced a high degree of stress than did Black participants, this does not explain the difference in cortisol levels between the two high CSS groups (choice A is wrong). If Blacks' physiological response to stress were more effective than that of Hispanics, one would expect abnormal cortisol levels to be present in the Hispanic group rather than in the Black group (choice B is wrong). Nothing in the passage suggests that pregnancy hormones improve HPA axis functioning. Further, even if this were true, this phenomenon would be a constant for all participants in the study and would not explain differences between groups (choice C is wrong). Answer = D

Oil drop, dropped above plates (see diagram on next side but don't look at answer) What single change to the experimental system would reduce the electric field strength necessary to maintain the oil drop in static equilibrium? A. Increasing the density of the oil B. Decreasing the distance between the plates C. Decreasing the mass of the drop D. Decreasing the charge magnitude on the drop

When the oil drop (with charge -q) is in static equilibrium, the electric force it feels, qE upward, is balanced by the gravitational force, mg downward. Thus, qE = mg, so E = mg / q. Choices A and D would necessitate an increase in E. Choice B would have no effect on E: don't be confused by the electric field strength in a capacitor, E = V / d. The question stem implies that the electric field strength will decrease, and is therefore not asking how that might happen. Moreover, decreasing d would increase E, not decrease it. The answer is C.

**The investigators transfected neuronal cells plated in small culture dishes and 48 hours later measured the levels of Aβ peptide. Which of the following procedures should be implemented to measure the levels of Aβ? peep diagram on next side A. Place the culture dishes under a microscope and directly count the amount of Aβ B. Harvest the cells, break down the cell membrane, and measure the levels of cellular Aβ using a specific antibody. C. Harvest the cells, break down the cell membrane, and measure the levels of cellular Aβ using PCR. D. Harvest the condition media from the dishes and use an ELISA (enzyme-linked immunosorbent assay) coated with anti-Aβ peptide to measure the levels of Aβ released. If want more graph/data analysis practice, can redo this passage 3/look it over if time

*TREND = When asked to decide which tecnhique to use to identify something, know what Western, Southern, and Northern blots do but also know characteristics of what you are trying to find ex. may say the protein is secreted so any option mentioning lysing cells to find protein is wrong Aβ peptide is released into the extracellular space (Figure 1 and Experiment 1) so it should be found in the condition media of the culture dishes. ELISA is the preferred method for secreted proteins found in the media, blood or extracellular space, therefore the harvested media can be used with an ELISA coated with anti-Aβ to measure the levels of Aβ (choice D is correct). Since the protein is secreted, any answer that suggests the measurement of cellular Aβ should be eliminated (choices B and C are wrong). Further, it is impossible to visualize and measure any protein or peptide with a microscope (choice A is wrong). Answer D

*It has been observed that when an alpha particle and a beta particle are given equivalent initial kinetic energies, the beta particle will travel a significantly greater distance in air before losing its energy. Of the following, which is the most likely explanation for this observation? A. The alpha particle's greater size and charge results in a greater number of interactions, which cause it to dissipate its energy faster. B. Beta particles have greater momentum. C. The beta particle's smaller size causes it to have more elastic collisions with air molecules, allowing it to travel farther. D. The greater velocity of the alpha-particle relative to the beta-particle results in an erratic, shorter mean free path for the alpha particle.

A. Choice B is false since the equation relating momentum, p, to kinetic energy, K, is p =square root of (2Km). Since the alpha and beta particles have the same K, the one with the smaller mass (the beta particle) has the lower momentum. Choice C is false since a smaller particle would tend to have fewer collisions, not more. Choice D can be eliminated because the greater mass of the alpha particle means it has a lower velocity than a beta particle of equal kinetic energy.The alpha particle would experience a greater electric force and a greater gravitational force than an equally-energetic beta particle, not because of its lower velocity, but because of its greater charge and its greater mass. [In fact, it can be shown the energy lost per unit distance for a particle of charge q and mass m moving through matter is proportional to ((q^2)m )/ K.] Choice A is correct and describes why alpha particles have shorter ranges than beta particles.

Based on information in the passage, which is the correct ranking of the melting points of compounds from lowest to highest? (diagram on next side, don't peep answer) A. 2 < 3 < 1 B. 2 < 1 < 3 C. 1 < 3 < 2 D. 3 < 2 < 1

A. For melting to occur, the intermolecular forces that hold the crystal together in the solid state must be overcome. The passage states that the dominant intermolecular forces in each compound are depicted in the graphics and quantified in the plots. From these plots it is clear that the crystal of compound 2 is held together by the weakest intermolecular forces, as the halogen-carbonyl bond only has an energy of around 4 kcal/mol. Therefore, compound 2 should have the lowest melting point (eliminate choices C and D). Whereas compound 3 shows one interaction with the single largest computed energy, the two intermolecular forces in compound 1 are greater when combined. Therefore, compound 1 will have the highest melting point (eliminate choice B).

Test subjects not adhering to instructions to limit salt intake to 6 grams per day likely demonstrated which of the following in testing data? A. Increased DPB due to the resulting increased osmotic pressure of the blood. B. Increased DPB due to the resulting decreased the osmotic pressure of the blood. C. Decreased DPB due to the resulting increased osmotic pressure of the blood. D. Decreased DPB due to the resulting decreased osmotic pressure of the blood.

A. High salt intake will increase the salinity of the blood which will draw water into circulation by osmosis, elevating the observed blood pressure (answer choice A is correct and choice C is incorrect). This principle is demonstrated by equation Π=iMRT in which i (essentially the number of dissolved particles) is directly proportional to osmotic pressure, Π. This can be considered a measure of the dissolved solutes and the pressure required to halt the osmotic passage of water seeking to equate molarities on either side of a membrane -- in this case, capillary walls in the kidneys, lungs, and peripheral tissues. If the blood is enriched in NaCl, its osmotic pressure will be elevated (eliminating choices B and D).

A group of scientists followed up the findings in Figures 1 and 2 by studying the impact of race on attraction. They found that after controlling for factors such as status, cultural similarity, and physical attractiveness, approximately 75% of respondents rated members of their own race and/or ethnicity as "more romantically desirable" than members of other races and ethnicities. Moreover, marriage data demonstrate that, as of 2010, approximately 92% of all marriages are between couples of the same race and/or ethnicity. This best demonstrates: A. prejudice in romantic desirability ratings and discrimination in marriage partner selection. B. discrimination in romantic desirability ratings and class inequality in marriage partner selection. C. class inequality in romantic desirability ratings and social exclusion in marriage partner selection. D. social exclusion in romantic desirability ratings and prejudice in marriage partner selection.

A. Prejudice involves biased opinions and thoughts about members of another group (such as a racial/ethnic out-group), but does not involve acting on those thoughts; since same-race mate preference involves an opinion about desirability, prejudice best explains the fact that approximately 75% of respondents rated members of their own race and/or ethnicity as "more romantically desirable" than members of other races and ethnicities. Discrimination, on the other hand, is prejudicial treatment of an individual by another individual based on perceived membership in a certain social group; same-race mate discrimination best explains the fact that approximately 92% of all marriages are between couples of the same race and/or ethnicity (choice A is correct; choice D is wrong). Class inequality refers to processes in society that limit a social group's status or opportunities for advancement; this does not best explain same-race romantic desirability ratings or marriage preferences. Similarly, social exclusion, or marginalization, describes social structures that leave certain groups at the fringes of society, rather than allowing them access to equal rights, opportunities, and resources; again, this does not best explain same-race romantic desirability ratings or marriage preferences (choices B and C are wrong).

*Retinal isomerase is used to alter the conformation of retinal. Retinal isomerase is most likely manufactured by: A. dehydration synthesis. B. hydrolytic synthesis. C. oxidation. D. glycosidic anabolism.

A. Retinal isomerase, an enzyme, is a protein, and proteins are synthesized by the condensation of amino acids. Condensation involves dehydration.

*The scientists expose their DNA aptamer to histone proteins with and without acetylation of the lysine residues at a pH of 7. When this mixture is placed at the center of an agarose gel and gel electrophoresis is run, the mixture separates into two bands. The DNA aptamer is found to be on the band: (One effect of acetylation in this context is that the charge of the otherwise-positive lysine residue is effectively neutralized.) A. closer to the positive terminal. B. closer to the negative terminal. C. at the center of the agarose gel. D. cannot be determined with the information provided.

A. The mixture of the histone proteins would separate into two separate bands by charge as both proteins are large and approximately the same size. Lys(Ac)-Histone H4 would be found closer to the positive terminal as the molecule is more negatively charged relative to the Lys-Histone H4 that lacks acetylation of the lysine residues. According to the passage, the DNA aptamer has a lower Kd and therefore higher affinity for the histone proteins with the lysine residues acetylated. Binding of the aptamer, which is an oligonucleotide and therefore negatively charged, to the histone would further increase the negative charge of the acetylated histone and result in a greater attraction to the positive terminal. As a result, the DNA aptamer would be found on the band closer to the positive terminal (choice A is correct; choices B, C, and D are wrong).

*The results of the manipulation check for the "no difference condition" are evidence of: (The results of the manipulation check indicated that, in the "no difference" condition, the vignette described a student who performed exceptionally well on mental rotation tasks -> the majority of boys guessed that the vignette described a boy, while the majority of girls guessed that the vignette described a girl.) A. in-group favoritism. B. out-group favoritism. C. a primary group. D. out-group discrimination.

A. The passage states that, in the "no difference condition," a majority of girls guessed that excellent performance was associated with a girl, while a majority of boys guessed that excellent performance was associated with a boy. An individual's positive bias towards his/her own group in known as "in-group favoritism" (choice A is correct; choice B is wrong). Primary groups are groups composed of members who interact frequently and on an intimate, long-term basis. Primary groups are typically composed of family members and very close friends; guessing a "girl" or a "boy" in the "no difference condition" is not an example of a primary group (choice C is wrong). While the students' responses on the manipulation check reflect a negative out-group bias, they are not considered an example of discrimination. The term discrimination describes when members of a group are denied access to resources or other privileges on the basis of their membership in that group (choice D is wrong).

Rank the following bonds in order of increasing polarity: I. C—Cl III. Cl—ClII. Si—Cl IV. P—Cl A. III < I < IV < II B. III < IV < II < I C. I < II < IV < III D. II < IV < I < III

A. The polarity of a bond depends on the difference in electronegativity between the two bonding atoms: The larger the difference, the more unequal the sharing of electrons, and the more polar the bond. Thus, the Cl—Cl bond is completely nonpolar, eliminating choices C and D. To find the final answer, look at the other extreme of the ranking problem for the remaining answer choices. Carbon is more electronegative than Si, therefore Cl, which is the most electronegative element depicted, will show the greatest electronegativity difference with Si. Therefore, of the four bonds, the Si—Cl bond should be the most polar (eliminate choice B).

Conducted in 1920, the Little Albert experiment consisted of conditioning a nine-month-old baby to fear a white, fuzzy stuffed animal by making a loud noise to startle the baby whenever the stuffed animal was presented. The purpose of the experiment was to see if a phobia could be induced in an otherwise normal, healthy baby. Based on this information, which of the following is true? A. If the baby was successfully conditioned to fear the white, fuzzy stuffed animal, then the stuffed animal, which started as a neutral stimulus, has become a conditioned stimulus. Correct Answer B. If, after Little Albert has been conditioned to fear the white stuffed animal, he also fears other white, fuzzy objects, like Santa's beard, this is known as discrimination. C. Little Albert's response to the loud noise is a conditioned response. D. The Little Albert experiment would be considered ethical today.

A. just read why B wrong (discrimination vs generalization) If Little Albert was successfully conditioned to fear the white, fuzzy stuffed animal, then it is true that the stuffed animal, which started as a neutral stimulus, has become a conditioned stimulus (choice A is correct). If, after Little Albert has been conditioned to fear the white stuffed animal, he also fears other white, fuzzy objects, like Santa's beard, this is known as generalization, not discrimination (choice B is wrong). A loud noise, which is startling to most organisms, including human babies, generates a natural fear response, which is an unconditioned response, not a conditioned one (choice C is wrong). These experiments would not be considered ethical today, partly because of the distress caused to the infant, partly due to the unknown nature of the outcome (will this baby forever fear white, fuzzy things?), and partly because infants are a well-protected subject population (choice D is wrong).

Just read over: After recovering from a severe illness caused by O157:H7, a person who could previously eat dairy products without difficulty has become lactose intolerant. Which of the following is the most likely cause of this? A. The disruption of the membrane during infection has also impacted the brush border enzymes. B. E. coli that normally reside in the large intestine are metabolizing the lactose and producing Vitamin K. C. O157:H7 is metabolizing the lactose. D. The toxin produced by O157:H7 changes the conformation of lactase.

Again, important to reference passage even if seems like memory question -> A. Lactose is a found in dairy products. It is a disaccharide consisting of glucose and galactose. Lactose is hydrolyzed by lactase, which is a brush border enzyme. The passage states that the infection by O157:H7 damages the epithelium; by doing so the brush border and the brush border enzymes are also damaged (choice A is correct). Without lactase, lactose cannot be digested and will enter the large intestine, where it will be metabolized by the E. coli that resides there, but this is the effect of lactose intolerance, and not the cause (choice B is incorrect). The person is recovering from the infection, so O157:H7 is no longer present (choice C is incorrect). There is nothing in the passage to suggest that the toxin produced by O157:H7 is a noncompetitive inhibitor of any enzyme or in any other way alters its protein structure (choice D is incorrect).

*All of the following are true of the sialylated receptor described in the passage EXCEPT: A. after being modified in the ER, the protein is transported to the plasma membrane. B. the signal recognition particle recognizes the fully-folded sialylated protein. C. translation of the protein begins in the cytosol. D. molecular chaperones located in the lumen of the ER may assist in correct folding of the receptor.

B. All proteins destined for the plasma membrane pass through the secretory pathway, including the sialic acid receptor described in the first paragraph. Posttranslational modification occurs in the ER before transportation to the plasma membrane via vesicles (choice A is true and can be eliminated). The signal recognition particle identifies partially translated mRNA-ribosome complexes with a targeting sequence in the cytosol and targets them to the rough ER, at which point translation continues and post-translational modifications (such as sialic acid) are added (choice B is false and the correct answer choice). Translation of all proteins, regardless of their cellular fate, generally begins in the cytosol (choice C is true and can be eliminated). Chaperones are often involved in assisting with protein folding in the ER (choice D is true and can be eliminated).

All of the following are true about photoreceptors in the retina EXCEPT: A. rods function best in reduced light. B. rods are highly sensitive to visual detail. C. cones process color information in bright light. D. cones are the only photoreceptor found in the fovea.

B. Although rods are more sensitive to lower levels of light than are cones, rods are not sensitive to detail and instead are generally involved in peripheral vision in normal or bright light (choice B is false and therefore correct). Due to their sensitivity to photons, rods do most, if not all, of their photoreception in dark situations (choice A is true and therefore wrong). Cones process color information in all settings, including bright light (choice C is true and therefore wrong). The fovea, the middle section of the retina on which images are generally focused, contains only cones (choice D is true and therefore wrong).

HIV, the virus that causes AIDS, is a retrovirus. Retroviruses have an RNA genome that, after entry into the host cell, is reverse transcribed to DNA, then incorporated into the host genome. RNA copies of the viral genome are produced using the normal host machinery, then packaged into viral capsid proteins for release. Which one of the following is most likely to contribute to the development of drug-resistant HIV? A. Mutation of the virus after insertion into the host-cell genome B. Frequent random errors in transcription by host-cell enyzmes C. Viral proteins folding differently in the presence of drug than in its absence D. Changes in the tertiary structure of viral RNA by drug

B. Drug resistance develops as mutant versions of the virus are produced and begin infecting new host cells. The change must be able to be passed on to viral progeny. If the viral proteins fold differently in the presence of drug than in its absence, this might account for the mechanism of action of the drug, but does not explain how drug resistance develops, nor could this be a heritable change (choice C is wrong). Changes in the tertiary structure of the viral RNA by the drug is another temporary, non-heritable change (choice D is wrong). Choices A and B are both plausible mechanisms for creating new, heritable versions of the virus. However, mutation of the virus after insertion into the host-cell genome would require that an error in DNA replication be made—a rather unlikely event, given the proofreading ability of DNA polymerase. Viral progeny are produced through transcription (creating a new RNA viral genome off the permanent DNA version inserted into the host-cell genome), and the question states that this occurs using the normal host machinery. Since the normal host RNA polymerases have no proofreading function, it is more likely that errors will occur here, leading to mutant virus (choice B is better than choice A).

According to instrumental learning theory, researchers' attempts to "train" subjects to do something would best increase the subjects' performance during the training phase by providing some sort of reward: A. on a fixed-interval schedule. B. on a variable-ratio schedule. C. for both correct and incorrect attempts. D. every ten minutes.

B. Instrumental conditioning studies show that variable-ratio schedules of reinforcement lead to the quickest increase in rate of desired responses (choice B is correct). Variable-ratio schedules provide a reward for an ever-changing quota of desired responses. The unpredictability inherent in this schedule prompts subjects to produce the desired response quickly and steadily in constant anticipation of reward. In contrast, a fixed-interval schedule provides reward after a fixed, or unchanging, period has elapsed. This schedule leads to a more gradual increase in the rate of desired responses (choice A is wrong). By definition, providing a reward every ten minutes is an example of a fixed-interval schedule (choice D is wrong). Rewarding incorrect attempts is inconsistent with instrumental learning theory, which maintains that rewards should be provided only for desired responses (choice C is wrong).

The following characteristics are shared between skeletal and smooth muscle with the exception of: A. Upstroke in action potential is secondary to inward Na+ current B. Potential exhibits a plateau C. Action potential opens voltage gated Ca2+ channels D. Ca2+ is the primary substrate for triggering contraction

B. Neither skeletal nor smooth muscle contraction is initiated by an action potential with a plateau; this feature is only characteristic of cardiac action potentials (answer choice B does not indicate a similarity and is the correct answer). The upstroke in the action potential for both skeletal and smooth muscle is triggered by an inward Na+ current (answer choice A states a similarity and is not the correct answer). Once received by either skeletal or smooth muscle, the action potential does, indeed, cause voltage-gated Ca2+ channels to open; in skeletal muscle, these channels are located on the sarcoplasmic reticulum while, in smooth muscle, they are located in the cell membrane (answer choice C is wrong). Indeed, Ca2+ is the molecular substrate for triggering contraction in both skeletal and smooth muscle; binding with troponin triggers skeletal muscle contraction and calmodulin binding activates myosin light-chain kinase in smooth muscle (answer choice D is wrong).

All of the following are components of plasma EXCEPT: A. fibrinogen. B. erythrocytes. C. potassium. D. immunoglobulins.

B. Plasma is the cell-free portion of blood. Proteins and salts are present in plasma, but cells—such as erythrocytes—are not, by definition.

The Whorfian hypothesis suggests that our perception of the world is based on our availability of language with which to describe it. If a language's only color words were white, black, red, and green, which of the following would a supporter of the Whorfian hypothesis believe? A. There is nothing purple in the environment of the native speakers of this language. B. Native speakers of this language perceive grass and the ocean as being the same color. C. It is evolutionarily important for native speakers of this language to identify the color red (e.g., to avoid poisonous berries). D. Native speakers of this language are most likely colorblind.

B. Since the Whorfian hypothesis states that language drives cognition, a supporter would believe that the only distinctions among colors that a native speaker of this language would perceive are among black, white, red, and green. Therefore, they would not see a difference between the green of the grass and the blue of the ocean (choice B is correct). Whorfian supporters would not necessarily believe that other colors are not present in this environment; the native speakers would simply accept that particular wavelength as one of the colors for which they have a term (choice A is wrong). The Whorfian hypothesis does not speak to why certain color terms are used as opposed to others, although there is a standard pattern described by Berlin and Kay (1969). It would therefore be irrelevant to the supporter whether or not the word red is an evolutionarily useful tool (choice C is wrong). If native speakers were colorblind and only used terms for the colors that they could see, this would contradict the Whorfian hypothesis, as cognition would be determining language and not the other way around (choice D is wrong).

A young man develops ophidiophobia, which is a fear of snakes. His fear is maintained because he intentionally avoids any encounters with snakes. Which of the following operant conditioning principles describes how the man is preserving his fear? A. Positive reinforcement B. Negative reinforcement Correct Answer C. Positive punishment D. Negative punishment

B. Since the avoidance of snakes is relieving the man's anxiety, he will continue to avoid snakes; a negative stimulus, the anxiety that the man feels from a snake's presence, is "removed" (or avoided) following his avoidance behavior. This pattern describes negative reinforcement (choice B is correct). The man is not adding a positive stimulus following the avoidance behavior (choice A is wrong). Punishment involves a decrease in a behavior. The man's decrease in anxiety is not going to lead to a decrease in the avoidance behavior (choices C and D are wrong).

There is a posited relationship between social capital and international giving. This might be explained by the fact that: A. wealth can increase one's ability to provide financial support internationally. B. networks can increase one's awareness of international causes. C. education can increase one's understanding of abstract concepts such as human rights. D. spirituality can increase one's connections to others, both domestically and internationally.

B. Social capital refers to the idea that social networks have benefits. It is possible that these networks could raise one's awareness of international causes, one of the three significant predictors of donations (choice B is correct). Wealth is an example of economic capital, education is an example of cultural capital, and spirituality is an example of spiritual capital (choices A, C, and D are wrong). Furthermore, while religiosity is referred to in the passage, and can, in fact, influence social networks and thus social capital, spirituality does not necessitate a relationship with a religious institution.

*During oxidative stresses, HbS can cause the affected red blood cells to sickle. The spleen is very vascular and is responsible for removing old or damaged red blood cells. Recurrent sickling can cause blockage of blood flow in the spleen, resulting ultimately in death of splenic cells. Which of the following would result from destruction of the spleen? A. Impaired ability to synthesize red blood cells B. Impaired ability to fight off encapsulated bacteria C. Impaired digestion D. Increased red blood cell destruction

B. The spleen is one of the major organs of the immune system. It is rich with lymphocytes, both B and T, and serves a major resource in the body's natural defenses against pathogens, especially encapsulated bacteria (choice B is correct). The spleen is not the site of red blood cell synthesis; the bone marrow is (choice A is incorrect). The spleen is not involved in digestion (choice C is incorrect). The spleen also removes damaged or old red blood cells. If the spleen is damaged, or infarcted, then there will be less destruction of red blood cells (choice D is incorrect).

Suzanne was in the middle of deep sleep and was suddenly jolted awake and alert by her alarm clock. Suzanne's brain wave state has most likely quickly shifted from: A. alpha wave to delta wave. B. delta wave to beta wave. C. theta wave to alpha wave. D. beta wave to theta wave.

B. This question calls for knowledge of four brain wave states. Beta waves are associated with alert wakefulness, while alpha waves are associated with relaxed wakefulness or drowsiness. Theta waves are active during drowsiness or sleep, and delta waves are associated with deep sleep. The question described Suzanne as being jolted from deep sleep to alert wakefulness, which corresponds to a shift from delta wave to beta wave activity (choice B is correct; choices A, C, and D are wrong).

If time: Positive cognitive biases in marital satisfaction forecasts would lead to: Passage: Theoretical perspectives offer three explanations of optimistic projections. The first perspective asserts that positive affective forecasting is an arguable strength because unrealistic optimism can cause cognitive processes to focus greater attention on positive experiences, subsequently helping to reduce the effects of negative experiences. The second perspective suggests that these projections are the result of random affective forecasting errors; for example, errors as a result of the failure to account for external factors on marital satisfaction. The third perspective proposes that positive marital forecasts are self-protective mechanisms that adopt an extended evaluative framework to allow for improvements in the presence of distress, particularly in relationships with high levels of investment (e.g., marriage). These opposing views will predict different outcomes in relationship trajectories. A. more severe declines in marital satisfaction, according to the first theoretical perspective. B. unpredictable changes in marital satisfaction, according to the second theoretical perspective. C. no changes in marital satisfaction, according to the second theoretical perspective. D. less severe declines in marital satisfaction, according to the third theoretical perspective.

B. This question tests one's understanding of the three theoretical perspectives presented in paragraph 2 and can be answered using the process of elimination. The first theoretical perspective suggests a positive correlation between predicted and actual marital satisfaction; positive forecasts are described as a strength, which can benefit, rather than harm, the relationship (choice A is wrong). The second theoretical perspective does not suggest a correlation between predicted and actual marital satisfaction; positive forecasts are described as random errors, and thus their effects on the relationship cannot be predicted (choice B is correct). Despite the fact that this perspective cannot predict specific changes, it does not predict the absence of changes (choice C is wrong). The third theoretical perspective suggests a negative correlation between predicted and actual marital satisfaction; positive forecasts are described as a form of self-protection when circumstances are stressful, which can harm, rather than benefit, the relationship (choice D is wrong).

*Creatine phosphate is a high-energy buffer that helps maintain the level of available high-energy phosphates such as ATP. During intense muscular exertion, creatine phosphate replenishes the muscle's ATP by releasing a relatively larger amount of energy in transferring its phosphate group to ADP. ATP is able to accept this phosphate group from creatinine phosphate and serve as a convenient carrier of energy because: A. it releases more energy upon hydrolysis than any other molecule in the body. B. it has phosphate groups with an intermediate transfer potential. C. it has such a specific and limited role. D. it is present in selected cells.

B. What makes ATP a particularly good energy carrier is that it has an intermediate transfer potential. This allows higher energy phosphate carriers (like creatine phosphate) to drive the synthesis of ATP by transferring their phosphate groups to ADP (choice B is correct). ATP's hydrolysis does not release the largest amount of energy of any molecule in the body, since creatine phosphate can drive ATP synthesis (choice A is wrong). ATP has a varied role in the body, as an energy carrier and also as a nucleotide for RNA and DNA (choice C is wrong), and ATP is present in every cell in the body (choice D is wrong)

*The episodic buffer allowed participants to: A. remember the instructions of the OLG task. B. form a visual picture of the video clip while recording response options. C. apply experience learned in prior games in order to generate options. D. avoid confusing the contents of one clip with those of another.

C. According to Baddeley, the working memory system is composed of four parts. The visuospatial sketchpad is a short-term visual store, the phonological loop is a short-term auditory store, the central executive is responsible for task-switching, and the episodic buffer integrates information from the other three systems and from long-term memory. An example of the contribution of the episodic buffer, therefore, is the recollection and application of prior game experience (choice C is correct). The episodic buffer is not directly responsible for the internal organization of experience (choice D is wrong). Rehearsal of verbal instructions would be the province of the phonological loop (choice A is wrong), while short-term visual memory of the video clips would be the province of the visuospatial sketchpad (choice B is wrong).

Which of the following best describes the expected differences between the G- and H-sites of GST? Just read over reasoning for right answer -> strategy ALL parts of choice have to be correct A. The G-site contains primarily polar amino acid residues whereas the H-site contains primarily nonpolar residues. B. Altering amino acid residues in the H-site will likely affect GST's Vmax, whereas altering those in the G-site will not. C. The G-site is more selective in substrate binding than the H-site. D. The G-site binds significantly smaller substrates than the H-site.

C. According to the passage, the G-site is responsible for binding GSH exclusively, whereas the H-site binds a wide variety of electrophilic substrates. Therefore, we expect the G-site to more selectively bind its substrate compared to the H-site (choice C is correct). Although the H-site is hydrophobic and should therefore contain primarily nonpolar residues, no such information is given about the G-site (choice A is wrong). While the passage states that substrate specificity is determined by changes in the H-site, altering important amino acid residues in the G-site will likely affect GST's ability to bind GSH and therefore affect the Vmax (choice B is wrong). The passage indicates that the G-site binds a relatively small tripeptide, GSH, but the comparative sizes of the electrophilic substrates bound by the H-site are not given (choice D is wrong).

*Which of the following is true for Reaction 2, once it is allowed sufficient time to achieve equilibrium? ΔG° = -30 A. ΔG < ΔG° B. ΔG = ΔG° C. ΔG > ΔG° D. ΔG = RT

C. At equilibrium ΔG = 0. Since reaction 2 has a negative value of ΔG°, it must be true that ΔG > ΔG°.

Sociologists refer to women's role as family caregivers in addition to their role as professionals as the "second shift". How might the "second shift" be interpreted as a psychosocial stressor? A. It increases women's sense of self-efficacy in the workplace. B. It results from prejudice against working women. C. It may lead to identity interference. D. It is a function of the "glass ceiling", which leads to a lower standard of living.

C. By definition, women's "second shift" refers to an ongoing duality of roles (the role of a working professional and the role of the family caregiver). When the demands of these roles conflict in terms of time and energy, tension results. Identity interference is the principle that states that people experience stress when two aspects of their identity are in conflict. Thus, the "second shift" could be understood as a cause of identity interference, which then constitutes a psychosocial stressor (choice C is correct). There is nothing to indicate that the "second shift" is the result of prejudice against working women (choice B is wrong). Self-efficacy is Albert Bandura's term for an individual's feelings of capability regarding the accomplishment of a given task; compounding one's roles would not increase self-efficacy in the workplace (choice A is wrong). The "glass ceiling" refers to unofficial limits on women's advancement in the workplace and does not directly relate to their additional roles outside the workplace (choice D is wrong).

Which of the following experimental findings would best confirm that a patient's type II diabetes was caused by a genetic defect and not a poor diet? A. Elevated patient blood glucose levels three hours after consumption of a concentrated glucose solution B. Discovery of an antibody in the patient's bloodstream that marks pancreatic beta cells for destruction C. Isolation of one of the patient's insulin receptors reveals a lysine-histidine substitution in the insulin-recognition site D. Detection of a virus that selectively targets and lyses insulin receptor expressing cells

C. Detection of an amino acid substitution in the insulin receptor insulin recognition site would be a strong indicator that a genetic malformation resulted in the production of an ineffective receptor (choice C is correct). Elevated blood glucose levels hours after consuming glucose is a hallmark of diabetes in general and does not indicate the specific cause of diabetes (choice A is wrong). The presence of an antibody that targets pancreatic beta cells is not indicative of type II diabetes but of type I diabetes (choice B is wrong). The presence of a virus that selectively targets insulin receptor expressing cells does not demonstrate that the cause of the patient's type II diabetes is a genetic defect (choice D is wrong).

The sound level of a sound wave that is striking an eardrum is increased from 35 dB to 55 dB. By what factor has the intensity of the sound increased? A. 2 B. 20 C. 100 D. 200

C. For each increase by 10 in the decibel level, the intensity of the sound increases by a factor of 10. In this case, the decibel level has increased by two units of 10 (since 55 - 35 = 20 = 10 + 10), so the sound intensity increases by two factors of 10, that is, by a factor of 102 = 100.

In glycolysis, how many moles of inorganic phosphate are consumed in the oxidation of 3 moles of glucose to pyruvate, as pictured in Figure 1? (Peep diagram on next side, but not answer) A. 1 B. 3 C. 6 D. 9 In another experiment, a mutase was used to transform the glycolytic intermediate 3-phosphoglycerate into 1-phosphoglycerate, which was immediately removed from the pathway and not able to become a substrate in subsequent steps. One would expect to find levels of ATP produced under these conditions to be: (use same diagram) A. higher in anaerobes than in aerobes. B. higher than levels produced by the arsenate-poisoned system. C. identical to levels produced by the arsenate-poisoned system. D. substantially greater than levels produced by the glycolytic pathway under normal conditions.

C. For every glucose molecule, 2 molecules of glyceraldehyde-3-phosphate are produced. Every molecule of glyceraldehyde-3-phosphate requires one molecule of Pi to be converted into 1,3-bisphosphoglycerate. For 3 moles of glucose, 6 moles of glyceraldehyde-3-phosphate are produced, requiring 6 moles of Pi. C. If 3-phosphoglycerate is converted into 1-phosphoglycerate, then no energy will be extracted as ATP later in the pathway by pyruvate kinase. Thus, the net ATP production per glucose will be 0 ATP. Per glucose, the ATP produced at each step is:-1 Hexokinase-1 Phosphofructokinase+2 Phosphoglycerate kinase0 Pyruvate kinase (normally +2)This is the same amount of ATP as in the presence of arsenate (choice C is correct and choice B is wrong). It will not increase the production of ATP (choice D is wrong). Whether the organism is anaerobic or not is irrelevant (choice A is wrong).

*Which of the following, if true, does NOT challenge the assertion that males' mental rotation advantage is purely biological in nature? A. There is no significant relationship between performance and endogenous levels of male sex hormones. B. There is a positive correlation between degree of men's confidence and performance. C. There is no significant difference on geometry test scores between girls taught by a female teacher and those taught by a male teacher. D. There is a negative correlation between girls' level of internalization of negative stereotypes and performance.

C. If gender differences in performance were attributable exclusively to biological factors, then exposure to a female role model proficient in mental rotation tasks would not be expected to affect the trend. Accordingly, no significant differences between girls taught by a female teacher and those taught by a male teacher would be expected (choice C does not challenge the assertion and is therefore correct). If biological differences alone accounted for trends in male versus female performance on mental rotation tasks, one would expect that the level of male sex hormones to be associated with greater performance on such tasks. If no such association were found, the notion that performance differences are due exclusively to biology would be called into question (choice A would challenge the assertion and is therefore wrong). A finding that another variable, such as degree of confidence or level of internalization of stereotypes, is correlated/associated with performance would suggest that gender differences may be attributable to factors other than biology (choices B and D would challenge the assertion and are therefore wrong).

In gastroesophageal reflux disease (GERD), gastric secretions exit the stomach via the lower esophageal sphincter and come in contact with the lower part of the esophagus. How would this affect the esophageal epithelium? Increased proliferation of the stratified squamous epithelium Conversion to columnar epithelium similar to that in the stomach Introduction of bicarbonate-secreting cells (The stomach epithelium mostly consists of simple columnar epithelium although these are not arranged in villi because no absorption of nutrients occurs in this organ. The small intestine is where absorption of nutrients occurs and its epithelium is specialized for this purpose. Simple columnar epithelium lines the villi, maximizing the surface area. The resulting brush border contains many enzymes essential for the hydrolysis of disaccharides and dipeptides, allowing for their absorption. Small bicarbonate-producing clusters of cells called Brunner's glands are embedded beneath the epithelium in the submucosa; secretions from these glands help to neutralize highly acidic gastric contents as they move from the stomach to the small intestine.) A. I only B. II only C. I and II only D. I, II, and III

C. Item I is true: in the short term, gastric acid would corrode the outermost layer of epithelium in the lower esophagus, increasing the production of cells (choice B can be eliminated). Item II is true: over time, the acid will destroy the epithelium and expose the submucosa. Columnar epithelium found in the stomach will gradually encroach on this area and become the dominant form of epithelium, since it is not sensitive to acid (choice A can be eliminated). Item III is false: There are specialized areas known as Brunner's glands in the proximal duodenum that produce bicarbonate to neutralize the acidic chyme as it passes from the stomach to the duodenum. However, there is no reason to assume that these cells would traverse the stomach to the lower esophagus (choice D can be eliminated and choice C is correct).

Which of the following scenarios best exemplifies modeling of deviant behavior? A. Because drug use is more common among cigarette smokers, smoking is considered a "gateway drug" B. Smoking is stigmatized both formally and informally in US society C. A teenager begins smoking cigarettes after repeatedly observing his older brother smoking D. Insurance companies charge a higher premium for smokers than for non-smokers

C. Modeling, according to social learning theory, occurs when an individual observes someone else's behavior and then behaves in a similar way. This is also known as observational learning or vicarious learning. If an individual observes his brother engaging in a deviant behavior (smoking), and then begins to engage in that behavior himself, this best demonstrates modeling of a deviant behavior (choice C is correct). While both drug use and smoking are generally considered deviant behaviors in society, this choice does not demonstrate modeling of deviant behavior (choice A is wrong). Deviant behavior, such as smoking, often is stigmatized both formally (through laws preventing smoking in certain places) and informally (through judgmental looks and comments from others) in US society, but this choice does not demonstrate modeling of deviant behavior (choice B is wrong). Similarly, insurance companies charging a higher premium for smokers could also be considered a form of stigmatization, but this choice does not demonstrate modeling of deviant behavior (choice D in wrong).

*Aβ peptides can negatively regulate various steps of acetylcholine (ACh) synthesis, release, and signaling. Which of the following would be the best target for pharmacological intervention? A. A drug that inhibits postsynaptic ACh receptors B. A drug that blocks voltage-gated Na+ channels C. An allosteric inhibitor of acetylcholinesterase D. A drug that inhibits choline reuptake by cholinergic neurons

C. Since the Aβ peptides negatively regulate ACh synthesis, release, and signaling, any kind of pharmacological intervention should be targeted at increasing ACh or its activity. An inhibitor of acetylcholinesterase (AChE) would prolong the life of ACh in the synaptic cleft, thus increasing its effect on the postsynaptic cell (choice C is correct). Drugs that inhibit postsynaptic ACh receptors would be a poor choice because they would inhibit ACh signaling (choice A is wrong). Blocking voltage-gated Na+ channels would have severe consequences throughout the body, as virtually all nervous system and muscular activity would be inhibited (choice B is wrong). Inhibition of choline reuptake by cholinergic neurons would reduce or prevent ACh synthesis (choice D is wrong).

*The following measurements were obtained by an investigator who observed a light ray that propagated across a boundary between air and Material X: speed of light in air = 3.0 × 108 m/s frequency of light in air = 5.0 × 1014 Hz angle of incidence = 30.0° angle of refraction = 23.5° density of Material X = 5.0 g/cm3 At what speed does light propagate through Material X? (Note: sin 23.5° = 0.4) A. 1.4 × 108 m/s B. 1.7 × 108 m/s C. 2.4 × 108 m/s D. 3.7 × 108 m/s

C. Snell's Law gives n1 sin θ1 = nX sin θX → (1) sin30° = nX sin(23.5°) → 0.5 = nX(0.4) → nX = 5/4. Since the index of refraction of Material X is 5/4, then, by definition, the speed of light is reduced by a factor of 5/4 from its speed in vacuum (or air). Thus, vX = (4/5)c = (4/5)(3 × 108 m/s) = 2.4 × 108 m/s. (Note that choice D can be eliminated immediately since it is greater than c = 3 × 108 m/s.)

TTF could accurately be described as: (TTF is an individual simply chooses the first solution that comes to mind.) A. a foundation for the representativeness heuristic. B. the result of syllogistic reasoning. C. a subtype of the availability heuristic. D. an indication of executive dysfunction.

C. The availability heuristic is a problem-solving strategy whereby the most salient information seemingly applicable to a problem is selected to produce a solution. For example, if people were asked whether there are more words with the letter "M" as the first letter than those with "M' as the fourth letter, they would likely respond that there are more words with "M" as the first letter; this is the case simply because such words are more easily recalled. Similarly, the TTF heuristic entails one's use of information that comes immediately to mind (choice C is correct). The representativeness heuristic is a strategy whereby a person chooses an option that appears most typical, or representative, of the solution. Such an approach is only possible when one is presented with multiple options, not when one must generate those options. Furthermore, this approach to problem solving is not a prerequisite to the TTF heuristic, which simply requires a person to select the first option that comes to mind (choice A is wrong). Syllogistic reasoning is a systematic process of logical thinking such that if A is related to B in X way, and if C is an example of A, then C will also be related to B in X way (choice B is wrong). While impulse-control problems may reflect a deficit in the executive system that regulates attention and behavior, the TTF heuristic is a strategy of deliberately choosing what comes to mind first, not a thoughtless or impulsive decision (choice D is wrong).

Question (may need to email) Which of the following is the most likely origin in the afferent sensory neuron of the α2δ-1 Ca2+ channels implicated in the inhibitory mechanism of GBP on signal transmission? A. Dorsal post-synaptic terminal B. Proximal pre-synaptic terminal C. Dorsal root ganglion D. Proximal portion of the axon

C. The dorsal root ganglion is the location of the neuron cell body where protein production occurs; thus, this is the most likely origin for channel proteins that would subsequently become embedded in the neuron cell membrane (choice C is correct). Since the afferent neuron begins in the periphery and ends in the CNS, the dorsal post-synaptic terminal of the afferent neuron would be located in the tissues from which the sensory signal is being sent making it a less likely source for channel proteins (eliminate choice A). As channel proteins are stored in vesicles in the proximal pre-synaptic terminal, it is likely that these proteins will be found here; however, the terminal itself is an unlikely origin due to the fact that protein synthesis typically does not occur in the terminal (eliminate choice B). Again, the proximal portion of the axon is a likely place to find channel proteins being transported to the terminal via anterograde transport by dynein but it is an unlikely origin (eliminate choice D).

Which of the following is the most likely origin in the afferent sensory neuron of the α2δ-1 Ca2+ channels implicated in the inhibitory mechanism of GBP on signal transmission? A. Dorsal post-synaptic terminal B. Proximal pre-synaptic terminal C. Dorsal root ganglion D. Proximal portion of the axon

C. The dorsal root ganglion is the location of the neuron cell body where protein production occurs; thus, this is the most likely origin for channel proteins that would subsequently become embedded in the neuron cell membrane (choice C is correct). Since the afferent neuron begins in the periphery and ends in the CNS, the dorsal post-synaptic terminal of the afferent neuron would be located in the tissues from which the sensory signal is being sent making it a less likely source for channel proteins (eliminate choice A). As channel proteins are stored in vesicles in the proximal pre-synaptic terminal, it is likely that these proteins will be found here; however, the terminal itself is an unlikely origin due to the fact that protein synthesis typically does not occur in the terminal (eliminate choice B). Again, the proximal portion of the axon is a likely place to find channel proteins being transported to the terminal via anterograde transport by dynein but it is an unlikely origin (eliminate choice D).

*An object with a height of 6 cm is placed at a distance of 20 cm from a concave mirror with a focal length of 12 cm. If the mirror produces a real, inverted image at a distance of 30 cm, the height of the image is: A. 4 cm. B. 6 cm. C. 9 cm. D. 12 cm.

C. The linear magnification m is given by the expression -i/ o. In this case, m = -(30 cm) / (20 cm) = -1.5, so the image is 1.5 times taller than the object. Therefore, if the object is 6 cm tall, then the image is 1.5(6 cm) = 9 cm tall. (Note: The minus sign on msimply signifies that the image is inverted, as stated in the question.)

Which of the following is NOT a component of Baddeley's model of working memory? A. Episodic buffer B. Long-term memory C. Multi-store model D. Phonological loop

C. The multi-store model was a model of memory developed by Atkinson & Schiffrin (1968). Baddeley's model of working memory was designed to improve upon the multi-store model's conception of short-term memory (choice C is correct). The episodic buffer mediates between the long-term memory and the central executive in Baddeley's model, and helps give chronological order to memories and stories (choice A is wrong). Long-term memory is a component of both Baddeley's model and the multi-store model (choice B is wrong). The phonological loop is a component of Baddeley's model. It helps to store and retrieve auditory memories and stimuli (choice D is wrong).

If the crystal structure of 1 indicates an approximate distance of 4.1 Å between the phenyl rings of adjacent molecules, which value best approximates the minimum amount of energy required to separate two molecules of the crystal? (diagram on next side, but don't look at answer) A. 4.2 kcal/mol B. 5.1 kcal/mol C. 9.3 kcal/mol D. 10.2 kcal/mol

C. The passage states that the potential energy curves in Figure 1 plot bond distance of the interaction between two molecules versus the energy of the interaction. The graph for compound 1 shows two interactions that must be broken to separate the molecules, π-π stacking and a hydrogen bond. At 4.1 Å the strength of the interaction between the two π-systems is approximately 5 kcal/mol (the minimum position of the graph). In addition, at a distance of roughly 3.5 Å the hydrogen bond is worth just over 4 kcal/mol. The summation of the two brings the total value to about 9 kcal/mol.

A stroke patient performs the task described in the passage. While he demonstrates awareness of the shapes 100% of the time, he is unable to produce speech in order to state which shape it is. Where did his stroke most likely occur? A. V1 B. LGN C. Broca's area D. Wernicke's area

C. The patient is able to see the shapes and follow directions but cannot verbalize what he is seeing; therefore, his stroke likely occurred in Broca's area, the part of the brain devoted to speech production (choice C is correct). His V1 is intact as demonstrated by his ability to correctly indicate when he is being presented with a shape (choice A is wrong), and the lateral geniculate nucleus (LGN) of the thalamus is not implicated in speech production (choice C is wrong). Wernicke's area is another language-related area of the brain, but a stroke there would lead to fluent aphasia, wherein the patient would be producing nonsensical speech. Were the damage to Wernicke's area, he also might not be able to comprehend speech and follow the directions for the task (choice D is wrong).

This study factored in perceptions of support as well as actual support. If both factors were found to be equally valuable in increasing the odds of survival, we might consider this outcome the most analogous to: A. a bimodal distribution. B. a type II error. C. the placebo effect. D. confirmation bias.

C. The placebo effect describes when the control group in a study receives a fake drug or treatment (one that is not meant to produce any changes or improvements in the subjects) and despite this, the subjects in the control group still experience/report changes or improvements. The perception of support is somewhat analogous to receiving a fake drug, and the fact that those who perceive support experience as much benefit as those who actually have support demonstrates a change or improvement that is most analogous to the placebo effect (choice C is correct). A bimodal distribution occurs in statistics when, instead of the characteristic one hump in a normal distribution, there are two distinct humps. While it is possible that some variable or outcome in this study might be bimodally distributed, there is not enough information in the passage or the question stem to reach this conclusion; therefore, this principle is not analogous to the finding that both real and perceived social support are equally valuable in increasing the odds of survival (choice A is wrong). In statistics, a type II error is the failure to reject a false null hypothesis; in signal detection theory, a type II error occurs when the signal is present, but the receiver fails to detect it (also known as a miss). Either way, a type II error is not analogous to the finding that both real and perceived social support are equally valuable in increasing the odds of survival (choice B is wrong). Confirmation bias occurs when an individual only looks for evidence to support his/her beliefs; this also is not analogous to the finding that both real and perceived social support are equally valuable in increasing the odds of survival (choice D is wrong).

The tertiary structure of proteins is partially due to disulfide bonds between cysteine residues. These bonds can be broken by treatment with a mild reducing agent as shown below: next side (don't look at answer) Which one of the following could be such a reagent? A. CrO3 B. H2SO4 C. NaBH4 D. CH3CH2OH

CrO3 is a good oxidizing agent (eliminate choice A) while H2SO4 (sulfuric acid) is a strong acid (eliminate choice B). CH3CH2OH (ethanol) is a poor nucleophile and has no reducing abilities (eliminate choice D). NaBH4 is a reducing agent since it has hydrogen present in the form of hydride. Answer is C

Which of the following could explain why a patient is blind in his left eye? A. His right V1 is irreparably damaged. B. His left optic tract is severed. C. His right optic tract is severed. D. His left optic nerve is severed.

D. Blindness in the left eye is different from blindness in the left visual field. Blindness in the left eye could only be due to damage to the eye itself or to the left optic nerve, the part of the visual pathway that spans from the retina to the optic chiasm (choice D is correct). Damage to either the V1 in the right hemisphere or the right optic tract would cause blindness in the left visual field, as these are parts of the pathway that receive information from both eyes about the left half of the world (choices A and C are wrong). If the left optic tract were severed, the individual would be blind in his right visual field but still receive information about the left half of his world from both eyes (choice B is wrong).

Ghrelin, the "hunger hormone," is released when the stomach is empty. It increases hunger and promotes secretion and motility in the digestive tract. The receptor for ghrelin is found on the same cells in the brain as the receptor for leptin. Which of the following is a true statement? A. The interaction of leptin and ghrelin can be considered competitive inhibition because they produce antagonistic effects. B. The interaction of leptin and ghrelin is considered to be allosteric inhibition. C. The interaction of leptin and ghrelin is considered uncompetitive because ghrelin only binds when leptin is already bound. D. There is no interaction between leptin and ghrelin because they operate on separate receptors.

D. Even though they bind to the same cells in the brain, the fact that leptin and ghrelin bind to separate receptors means that there cannot be any kind of inhibition or interaction between them (choice D is correct). For competitive inhibition to occur, two things must be trying to bind to one site; for example, if both leptin and ghrelin bound to the same site on the same receptor. The fact that they have separate receptors prevents this (choice A is wrong). Likewise, allosteric inhibition requires that the inhibitor bind to a separate site on the same enzyme or receptor (choice B is wrong), and there is nothing in the question stem or passage to suggest that ghrelin can only bind when leptin is bound (choice C is wrong).

A scientist wants to test the heritability of sexual attractiveness in college-age females. This could be best accomplished by: A. using monozygotic twin studies to measure the percentage of sexual attractiveness caused by genes. B. using dizygotic twin studies to measure the percentage of sexual attractiveness caused by genes. C. using dizygotic twin studies to measure the percentage of variation in sexual attractiveness attributable to genetics. D. using monozygotic twin studies to measure the percentage of variation in sexual attractiveness attributable to genetics.

D. Heritability is best tested with monozygotic twin studies, since monozygotic twins share the same genes. Also, heritability is defined as the proportion of observable differences in phenotype attributable to genetic causes (choice D is correct). Heritability does not measure the percentage of an observable trait caused by genes, but only has meaning within the context of proportion attributable to variation (choice A is wrong). The other two answer choices can be eliminated because heterozygotic twin studies do not measure heritability as well as monozygotic twin studies do (choices B and C are wrong).

It was observed under the microscope that a small section of bone consisted of concentric rings of fibrous material surrounding an open circular region. The bone section was identified as an osteon. The circular region is expected to contain: A. mature osteocytes. B. maturing chondrocytes. C. large blood cell precursors. D. blood vessels.

D. In compact bone, the osteon is the basic unit of structure. Concentric rings surround the central circular region that contains blood or lymph vessels (choice D is correct). Osteocytes can be found in small spaces along channels branching away from the central channel (choice A is wrong). Bone is the site of the marrow in which blood cell precursors are found, but bone marrow is found in spongy bone and not in compact bone where osteons are found (choice C is wrong). Chondrocytes are found in cartilage (choice B is wrong).

Can just read reasoning over bc would also need whole text: Which of the following independent clinical findings would most strengthen the investigators' results? A. Postmortem analysis of brains from Alzheimer's disease patients showed reduced levels of total Numb mRNA. B. Postmortem analysis of brains from Alzheimer's disease patients have endosomes that stain positive for Numb. C. Alzheimer's patients show defects in the alternative splicing machinery and the γ-secretase activity. D. Brains from transgenic mice that exclusively express the L-Numb isoform showed reduced formation of neuritic plaques compared to controls.

D. Since the investigators were specifically looking at the role that Numb isoforms might play in APP trafficking, the correct answer should include data that address the role of the Numb isoforms, rather than general statements on Numb. The only answer choice that does this is choice D. L-Numb reduces Aβ which has been shown to cause neuritic plaque formation (see the first paragraph). Therefore, reduced neuritic plaque formation in mice that selectively express L-Numb would specifically support the investigators' data (choice D is correct). Reduced levels of Numb mRNA is a general observation that does not provide any information on the specific functions of the Numb isoforms in the brain (choice A is wrong). Numb is found in the endosomes of all cells, regardless of their disease state (see the third paragraph, choice B is wrong). General defects in the splicing machinery could potentially affect hundreds of protein isoforms (not just Numb); further, the activity of γ-secretase is unrelated to Numb function (choice C is wrong).

*Margaret was more afraid of dying in a terrorist attack than dying from dangers lurking in her own neighborhood because she was watching so many news reports of deaths by terrorist attack. This is an example of what psychological construct? A. Normative influence B. Fixation C. Attribution theory D. The availability heuristic

D. The availability heuristic states that a person will make a decision based on the information most readily available in memory. If Margaret has been watching many reports of terrorist dangers on the news, she will be more likely to worry about those than the dangers lurking in her own neighborhood about which she has heard little (choice D is correct). Normative influence refers to a group effect that results from individuals' desire to be liked and to win the approval of others, which does not explain Margaret's fear of terrorist attacks (choice A is wrong). Fixation refers to a state wherein an individual is attached to things or behaviors more appropriate to an earlier stage of psychosexual developmental (choice B is wrong). Attribution theory describes a social-cognitive approach to creating cause-and-effect relationships. Informational influence means that she is using the information in an attempt to be correct. That is not her motivation; rather, she is creating cause-and-effect connections between the information she hears and the decisions she makes. She is not worried about being right (choice C is wrong).

*A woman spoke to a behaviorist about the development of her pteromerhanophobia, which is a fear of flying. She stated that when she was a young child she would start crying when she heard loud noises. Accordingly, every time that she was in a plane she began to cry from the roar of the engines. Soon any time that she saw a plane she felt immense fear. Which of the following is the conditioned stimulus that led to her specific phobia? A. Loud noises B. Crying C. Engines D. Planes

D. The conditioned stimulus is paired with the unconditioned stimulus repeatedly and, over time, can elicit a conditioned response. In this case the plane was the conditioned stimulus that was paired with the unconditioned stimulus, loud noises (choice D is correct). The loud noise, which is produced by the engine, is the unconditioned stimulus (choices A and C are wrong). Crying is the response to both the unconditioned stimulus and the conditioned stimulus. Therefore, it could be either the unconditioned or conditioned response (choice B is wrong).

Which one of the following expressions is equal to the pressure difference between Point c and Point d? A. ρgasgv B. ρgasgy C. ρliquidgv D. ρliquidgy

D. The difference in pressure between two points in a static fluid is given by the expression ρfluidgy, where y is the vertical distance between the two points. (Note that choices A and C can be eliminated, because the units are wrong: density times acceleration times velocity does not give pressure.)

*The iron law of oligarchy: A. asserts that, because of the constant struggle between the labor class and the ownership class, democracy will be inevitably replaced by socialism. B. explains how formal social norms are codified into laws. C. describes the process whereby tasks are broken down into component parts to be completed by different workers in an organization. D. suggests that democracy is not possible in large groups or organizations.

D. The iron law of oligarchy is a theory in sociology that states that regardless of how democratic an organization is, all organizations will eventually and inevitably end up with a majority of the power in the hands of a few, thus leading to an oligarchy. The iron law of oligarchy suggests that democracy is practically and theoretically impossible, particularly in large and complex organizations (choice D is correct). Karl Marx asserted that, because of the constant struggle between the labor class and the ownership class, capitalism (not democracy) will be inevitably replaced by socialism (choice A is wrong), but this has nothing to do with the iron law of oligarchy. The iron law of oligarchy does not explain how formal social norms are codified into laws (choice B is wrong). Rationalization, which is a major component of bureaucracy, describes the process whereby tasks are broken down into component parts to be completed by different workers in an organization (choice C is wrong).

Which of the intermolecular bonding interactions described above is the least rigid (the least difficult to displace from its equilibrium length)? A. Hydrogen bond in 1 B. Stacking interaction in 1 C. Halogen-carbonyl bond in 2 D. bond in 3

D. The stiffness of a bond can be determined from the potential energy plots given in the figure. Stiffness is a measure of how much energy must be applied to change the bond length. A very narrow dip in the potential energy plot means that stretching the bond a little requires a lot of energy. This is different than the bond energy, which is indicated by how deep the dip in the plot is. As seen in the plot for compound 3, this bond has a relatively wide dip, meaning that application of only a little energy can change the bond a good deal. Therefore, choice D is correct. From another perspective, use a Sesame St. strategy. The graph that looks most different from the others is the widest one, so if you had to take a guess, the one for compound 3 would be the best one to choose.

*Review orgo -> If the students had used propylamine in Experiment 2 in place of piperidine, what functional group would they synthesize in Step 1? see diagram on next side A. Enamine B. Amide C. Imine Correct Answer D. Lactam

Enamines form from the combination of an aldehyde or ketone with a secondary amine (like piperidine). Since propylamine is a primary amine (eliminate choice A), an imine is formed instead (choice C is correct). Amides are carboxylic acid derivatives and form from the reaction of an amine with any other derivative (eliminate choice B). A lactam is a cyclic amide, so choices B and D are effectively the same answer (eliminate choice D). Answer is C

*Good analysis practice -> Which of the following might be a plausible explanation for the data observed in Figure 3? Peep graph on next side but don't peep answer A. GBP provides definitive symptomatic control for mechanical allodynia. B. Increasing expression of Ca2+ channels occurs gradually over weeks after an injury and GBP appropriately dosed may provide effective symptomatic control. C. The GBP dose administered was appropriate to control mechanical allodynia in this mouse for the time during which it was administered. D. Both ipsi- and contra-lateral paws demonstrated response to GBP during the time administered.

Figure 3 demonstrates that GBP only provides symptomatic control and likely needs to be administered with an increasing dose as is suggested by the negative slope of the threshold of the medicated ipsilateral trace during the time in which medication was administered (choice B is correct). The drop in threshold after ceasing GBP indicates that treatment with GBP only provides temporary symptomatic control, rather than definitive resolution of the symptoms (eliminate choice A). The negative slope of the threshold curve during treatment implies that the GBP dose was insufficient to keep up with increasing allodynia, likely due to increasing Ca2+ channel expression per the mechanism discussed in the passage (eliminate choice C). Only ipsilateral paws responded to GBP treatment over this timescale as demonstrated by a lack of threshold decrease for contralateral paws (eliminate choice D). Answer is b

Half maximal inhibitory concentration (IC50) is the concentration of an inhibitory compound required for 50% inhibition of a biological process (such as infection) in vitro. Which of the following is true of HAd37's interaction with ME0322? (see figure on next side, don't peep answer) A. The IC50 of ME0322 is approximately 5 μM, so ME0322 is a less effective inhibitor of HAd37 than sialic acid. B. The IC50 of ME0322 is approximately 5 μM, so ME0322 is a more effective inhibitor of HAd37 than sialic acid. C. The IC50 of ME0322 is approximately 5000 μM, so ME0322 is a less effective inhibitor of HAd37 than sialic acid. D. The IC50 of ME0322 is approximately 5000 μM, so ME0322 is a more effective inhibitor of HAd37 than sialic acid.

Figure 3 shows inhibition curves for ME0322 and sialic acid. The inhibition curve of ME0322 is left-shifted relative to the inhibition curve of sialic acid, indicating more activity per unit inhibitor (choices A and C can be eliminated). According to the question, the IC50 can be found by determining the concentration of compound at which 50% of a biological process (in this case, viral infection) is inhibited. For ME0322, this occurs between 0.001 and 0.01 mM (see Figure 3). Since there is a 1000-fold difference between mM and μM, this equals a concentration between 1 μM and 10 μM (choice D can be eliminated and choice B is correct).

Use diagrams on next side of card: Which one of the following graphs best illustrates the relationship between a, an oil drop's acceleration perpendicular to the plates, and E, the strength of the electric field between the plates?

For these types of questions see what the variables are and put together equations you know (or ones given, or combine both, but think about what you know background knowledge first) to make a y = mx + b or quadratic, etc and see what graph it matches The net force on an oil drop (of charge -q) between Plates A and B is the difference between the electric force and the gravitational force: Fnet = qE - mg. Since Fnet = ma, the acceleration of the oil drop is given by the equation a = (q/m)E - g. This is an equation of the form "y = mx + b," whose graph is a straight line. The answer must therefore be graph C.

Great thinking problem: what should I do when I see one like this?: Neuraminidase is an enzyme which selectively cleaves sialic acid derivatives at their point of attachment to a protein. Which finding, when combined with the data from Experiment 1, is most likely to lead researchers to conclude that GD1a is NOT the main mediator of HAd37 (Human Adenovirus) entry into HCE cells (human cells)? Background: sialic acid derivative are on human cell membranes and can be used by HAd37 to infect cell and dude wanted to see which derivative type was used the most by the HAd37. He did experiment neutralizing different sialic acid derivatives and observed how much infection there was for each group -> see next side for experiment data, don't peep answer A. When treated with neuraminidase, HAd37 viruses do not exhibit decreased infectivity against HCE cells. B. When treated with neuraminidase, HCE cells are still infected by HAd37 at the same levels as in the anti-GD1a treatment. C. When treated with neuraminidase, HCE cells are still infected by HAd37 at the same levels as untreated cells. D. When treated with neuraminidase, decreased binding of anti-GD1a to HCE cells is observed.

For these types of questions, draw out what's happening to those receptors for each answer choice and see if it makes sense The question introduces the concept of neuraminidase, an enzyme which, when applied to cells, removes sialic acid derivatives such as GD1a from the surface of the cell. Treating a cell with neuraminidase should make it impossible for HAd37 to infect the cell if HAd37 does so via a sialylated receptor. Since the sialic acid molecules are found on the host cells and not on the viruses, treating HAd37 viruses with neuraminidase should have no effect on infection (choice A is wrong). If neuraminidase-treated cells are still infected by HAd37 at the same level as anti-GD1a treated cells this would support the researchers' idea that GD1a is the main mediator of Had37 infection (both neuraminidase and anti-GD1a have the same effect of removing GD1a from the cell surface, choice B is wrong). However, if cells treated with neuraminidase (i.e., lacking GD1a) are still infected by HAd37 at the same level as the non-treated cells (i.e., that have GD1a), it suggests that GD1a is not critical for infection (choice C is correct). Because the neuraminidase cleaves sialic acid derivatives, one would expect there to be a lower level of binding of anti-GD1a to neuraminidase-treated cell surfaces. However this does not support or disprove the researchers' hypothesis (choice D is incorrect). Answer is C

Based on the data in Figure 1, which of the following is true regarding the members of groups A and B? See graph on next side A. There was a greater concentration of acetylcholine in the cardiac muscles of the group A members than was the case for the group B members. B. The vagus nerve was active for the group B members, but inactive for the group A members. C. Group A members' sympathetic nervous systems were activated when they re-entered the shopping mall. D. The hypothalamus was stimulated more in group A members than in group B members.

Group A members had a lower BPM when they re-entered the shopping center than did the individuals of group B. Acetylcholine has an inhibiting effect on cardiac muscle and therefore would cause a decrease in heart rate. Thus, group A members would have a greater concentration of acetylcholine present around their cardiac tissue since they had a lower heart rate (choice A is correct). When activated, the vagus nerve causes a decrease in heart rate and is part of the parasympathetic nervous system. Therefore, group A members' vagus nerves would be more stimulated (choice B is wrong). Sympathetic nervous system activation causes "fight or flight" symptoms. Therefore, it would cause an increase in heart rate and would be expected more for the members of group B (choice C is wrong). Hypothalamic stimulation causes an increase in heart rate, as well as an increase in perspiration. Therefore, it would be expected that the group B members would experience greater hypothalamic stimulation (choice D is wrong). Answer is A

According to the mechanism shown in Figure 1, would the function of the B-Raf kinase domain be eliminated by the substitution of Glu for Asp576? (Peep next side slowly to see diagram but don't look at answer) A. No, Glu can bind the divalent metal cation chelated by ATP in step 1. B. No, Glu can activate the nucleophile shown in step 2. C. Yes, Glu is too weak a base to abstract the hydroxyl proton shown in step 1. D. Yes, Glu would destabilize the transition state shown in step 2.

In Figure 1, Asp576 activates the nucleophile shown in step 2 by deprotonation of a hydroxyl group. Glutamate, another acidic amino acid with a slightly longer R-group would be able to function similarly in the same position in the B-Raf catalytic loop. Substitution of Glu for Asp576 should then have a limited effect on the enzyme function. Asp576 is not shown to interact with magnesium in Figure 1 (eliminate choice A). In addition, the glutamate would have a similar basicity as the aspartate, making choice C incorrect. B. IS CORRECT

*Theorists using the conflict perspective to address questions of health, illness, and medicine are LEAST likely to consider: strategy for this? A. the commodification of health care delivery. B. the presence of disparities in health and health care. C. the power of professionals in patient-provider relationships. D. the dependence of medical institutions on profit.

See if any of the answer choices resembles another foundational sociological perspective C. Each of the main sociological perspectives—structural functionalism, conflict theory, and symbolic interactionism—is expected to approach health and health care differently. The conflict theory approach considers the competition for limited resources. According to this theory, limited resources, a negative result of capitalism, cause inequalities and power differentials in society. This concept can be extended to the medical institution. Furthermore, Karl Marx, the founding conflict theorist, focused on the internal tensions produced in capitalistic societies. Thus, it is expected that the conflict perspective would consider the commodification of health care and the dependence of institutions on profit (choices A and D can be eliminated). Health care can thus be purchased and sold in a marketplace, which allows those with resources and influence to control pertinent decisions, thus maintaining inequalities. Furthermore, these social inequalities determine the distribution of health care access and resources. Thus, it is expected that the conflict perspective would consider the resultant disparities in health and health care (choice B can be eliminated). However, while conflict theorists are interested in the creation and maintenance of power differentials, as proponents of a macro-sociological perspective, this interest is applied to social structures as a whole. The patient-provider relationship would perhaps be of more interest to symbolic interactionists, as symbolic interactionism is a micro-level perspective focused on the transfer of information through communication between individuals (choice C is not an expected focus of conflict theorists and is the correct answer). Furthermore, it is possible that a power differential is inherent in the provider-patient relationship as a result of the expertise required for medical professionals, thus making an egalitarian relationship difficult to create.

Based on the passage and the data in Figures 1 and 2, a heterosexual male in a high-traffic area such as a party who wants to be seen as more desirable should: (just go over the strategy of what you would look for in the graphs on next side) A. assess physical characteristics such as facial symmetry and skin complexion. B. approach females directly who are also non-stationary, offering more chances for interactions. C. remain seated and allow others to approach him. D. approach stationary females directly.

Strategy: look for highest bar for woman's rating of man (highest light gray bar) and see that woman approaching man seated had the highest woman rating. The results depicted in Figures 1 and 2 show that by remaining seated and waiting for others to approach, both males and females were rated as more desirable; therefore, a heterosexual male partygoer who wants to be seen as more desirable should remain seated and allow others to approach him (choice C is correct). The assessment of physical characteristics such as facial symmetry and skin complexion is mentioned in the passage, as males tend to assess these traits in female potential mates; however, this assessment does not have an impact on the desirability of the male himself (choice A is wrong). The data from the figures show that approaching others resulted in a lower desirability rating for the individual (choice B and choice D are wrong). Answer is C

**(GO OVER) Benzaldehyde can be oxidized in air to benzoic acid, a common laboratory impurity, as shown below. (see next side) A diethyl ether solution of benzaldehyde containing benzoic acid impurities may be purified by extraction with which of the following solutions? I. 10% NaHCO3(aq) II. 1.0 M HCl(aq) III. 1.0 M NaOH(aq) A. I only B. I and III only C. II and III only D. I, II, and III

This is basically describing how only bases I and III will react with one of the compounds (the acid) so that's how you purify it B. In this Roman numeral question, eliminating choices that do not show differential solubility for benzaldehyde and benzoic acid is the best approach. Both benzaldehyde and benzoic acid are water-insoluble, so extractions that do not appreciably change the structure of one of the two will not allow effective separation. Because benzoic acid is acidic it is deprotonated by NaHCO3, yielding the water soluble salt, sodium benzoate. Since a salt is soluble in aqueous environments, item I is true (eliminate choice C). Neither benzaldehyde nor benzoic acid will react with or show differential solubility in the presence of the strong acid HCl, so both will remain dissolved in diethyl ether (eliminate choice D). In a similar fashion to using the weak base above, extraction with aqueous NaOH will induce salt formation by deprotonating the carboxylic acid (eliminate choice A).

**One effect of acetylation in this context is that the charge of the otherwise-positive lysine residue is effectively neutralized. The aptamer designed by the scientists binds selectively to histone proteins whose surrounding DNA: check next side for diagram, don't peep answer A. is bound less tightly than the DNA of histone proteins without acetylation of the lysine residues. B. is bound more tightly than the DNA of histone proteins without acetylation of the lysine residues. C. is acetylated. D. is not acetylated.

Tip: don't forget to use charts and graphs given to you!!! According to the passage, the aptamer has a lower dissociation constant and therefore a higher affinity for the histone proteins with the lysine residues acetylated. In order for the aptamer to bind the histone, the normal DNA around the histone must be relaxed enough to give the aptamer access to the histone. Since DNA is negatively charged, it would bind more tightly to histone proteins when the lysine residues are positively charged (in other words, when the histone is NOT acetylated). When acetylated, the lysine residues are no longer charged so the normal DNA around the histone protein would be bound less tightly, and the aptamer can get access to the histone and bind to it (choice A is correct and choice B is eliminated). There is no mention of DNA being acetylated in the passage (choices C and D are wrong). Answer is A

*In the alkylation step of Experiment 2, the carbon β to the nitrogen acts as: A. a nucleophile due to partial negative charge from resonance. B. a nucleophile due to partial negative charge from induction. C. an electrophile due to SN2-type reaction. D. an electrophile due to an inductive effect. What do we do to determine if something will be a nucleophile or electrophile?

To determine if something will be a nucleophile or electrophile, draw resonance, if see neg charge produce somewhere then that means it's a good nucleophile The carbon β to the nitrogen acts as a nucleophile, because it reacts with methyl iodide, which is an electrophile; this eliminates choices C and D. The partial negative charge on the β carbon is due to resonance, as can be seen most easily by analogy to an enolate ion. Answer is A

**The screening process of SELEX for high affinity oligonucleotides incorporates which of the following? A. Size-exclusion chromotography B. Gel electrophoresis C. Ion-exchange chromatography D. Affinity chromatography

Trick -> think about what would have to be different b/w these molecules for these techniques to work, and then remember all these molecules are the same size, etc D. The second and third step of SELEX involves selection of oligonucleotides that bind specifically to the target protein, which is affinity chromatography (choice D is correct). As stated by the passage, all oligonucleotides in the library have the same length. As a result, they cannot be separated by size or charge (choices A, B, and C are wrong).

T/F -> the following statement is an example of the dramaturgical approach: Medical professionals use impression management to manage their presentations and influence the perceptions of others, such as their colleagues.

True: The dramaturgical perspective suggests the use of impression management in most situations, including the specific example of health care settings. Impression management, also referred to as self-presentation, is the conscious or unconscious process of managing one's public image, as described above. For example, medical professionals might use impression management at the hospital in order to present themselves as knowledgeable health care providers who are active in the treatment of patients


Ensembles d'études connexes

Renal, Urinary, and Reproductive Systems Adaptive Quizzing

View Set

Nurs. 120 - Ch. 9 Cultural Awareness

View Set

ORGANIZATIONAL THEORY AND BEHAVIOR TEST II

View Set

chapter 13a. What Is a Psychological Disorder?; Anxiety Disorders, OCD, and PTSD

View Set

Pregnancy, Labor, Childbirth, Postpartum - At Risk

View Set

Explanations from Incorrect EAQ Questions

View Set

Consumer Behavior Chapter 2 quiz questions

View Set

Chapter 7 & 8 Guided Reading Review

View Set